LSAT and Law School Admissions Forum

Get expert LSAT preparation and law school admissions advice from PowerScore Test Preparation.

User avatar
 Dave Killoran
PowerScore Staff
  • PowerScore Staff
  • Posts: 5853
  • Joined: Mar 25, 2011
|
#22744
Complete Question Explanation

Flaw in the Reasoning-SN. The correct answer choice is (E)

One of the nice things about Flaw questions is that their answers can often be prephrased—and if you didn't happen to notice flawed reasoning, the question stem provides a reminder. In this case, the flaw is a very specific one. The author says second sentence in the stimulus that achieving the goal will require three things:

  • 1. A knowledge of neurons and their basic functions (N)
    2. A knowledge of how neurons interact (I)
    3. A delineation of the faculties to be explained (D)
This is conditional reasoning, which can be diagrammed as follows:

  • E = explain mental functions in neurobiological terms

    N = knowledge of neurons and their basic functions

    I = knowledge of how neurons interact

    D = delineation of psychological faculties


    ..... ..... N
    ..... ..... +
    E :arrow:           I
    ..... ..... +
    ..... ..... D

At present, the author states that we know a lot about #1 (N) and #3 (D). The author then suddenly concludes that we're bound to be headed for complete neurobiological explanation soon (E). But what is missing? In this case, the author didn't provide any information regarding what we know about #2 (I).

In more abstract terms, this reasoning is flawed for two reasons. First, no information regarding I has been established, so there is a big hole in the information we have about this relationship. Second, information about N and D occurring (and even I if we had it) cannot be used to establish that E occurs as this would constitute a Mistaken Reversal. The question stem asks you to identify the flaw in the method of reasoning and so you should seek an answer that trades on one or both of the prior points.

Answer choice (A): This is not an internal contradiction because the physicalists claimed that everything would ultimately be explainable and the conclusion states the phsyicalists are correct.

Answer choice (B): The author has the right to note that simply that there is a "substantial amount of fundamental knowledge about the basic functions of neurons" and is not required to go into specific details about what that encompasses.

Answer choice (C): This is not the case. A distinction is noted in the first sentence where "all mental functions will be explainable in neurobiological terms," and then the argument later refers to mental functions. There's no conflation of terms.

Answer choice (D): The author doesn't need to do this. The argument simply describes what we know about the situation; a value discussion isn't needed to make the argument valid.

Answer choice (E): This is the correct answer. The stimulus, as discussed above, does not provide any information about how neurons interact (I), and thus this answer choice describes an error in the reasoning.
 reop6780
  • Posts: 265
  • Joined: Jul 27, 2013
|
#12093
The stimuli describes what factors should be required to achieve the goal.

And, I guess the way writer did not include any information about "how neurons interact" was the problem?

Is this kind of flaw included among the flaws presented in LR bible?
User avatar
 Dave Killoran
PowerScore Staff
  • PowerScore Staff
  • Posts: 5853
  • Joined: Mar 25, 2011
|
#12098
Hi Hyun,

Yes, that flaw is both an Error of Conditional Reasoning and an Error in the Use of Evidence, but one so obvious as to not be explicitly covered. When they say three things are required and then they only mention two, it's pretty clear a mistake occurred and that the argument has faulty construction.

Please let me know if that helps. Thanks!
 reop6780
  • Posts: 265
  • Joined: Jul 27, 2013
|
#12138
Thanks, Dave !
 starre
  • Posts: 28
  • Joined: Sep 27, 2016
|
#30378
I selected A for this question, and I'm still confused as to why A is wrong. I understand why E is right, but A also seems write to me. I have the first sentence, after the word "all", diagrammed as MF-->NB with MF=mental functions and NB=neurobiological terms. I have the last sentence, after the word "thus", diagrammed as NB-->MF. Therefore, I see a mistaken reversal, as the first poster here had, which I thought was answered by A. Could you explain why A is wrong? Did I diagram those parts incorrectly? Thank you in advance!
 Adam Tyson
PowerScore Staff
  • PowerScore Staff
  • Posts: 5153
  • Joined: Apr 14, 2011
|
#30399
Thanks for the question, Starre! I'm going to direct you to the discussion about to see how this question should be diagrammed, and point out that it's about knowledge - we need to know certain things if we are to explain certain things in a certain way. The ability to explain is the sufficient condition and the three points of knowledge are the necessary things.

You are correct that there is a Mistaken Reversal going on, and as was discussed earlier in this post that error is not the only error and is not the one that the correct answer focuses on. In fact, none of the answer choices bring it up. So what's going on in answer A?

A might describe an internal contradiction, a case where the conclusion is in direct conflict with the premises. If my premise is conditional, if A then B (A --> B), and then I say that A happened and conclude that B did NOT happen, that would be an internal contradiction - my premise and my conclusion are at odds with each other. But that's not the same as a Mistaken Reversal. A Mistaken Reversal is where you reverse the order of the conditional claims, and the problem with that is that you cannot PROVE it to be true based on the original claim. It COULD, however, BE true - Mistaken Reversals and Mistaken Negations are possible, but not provable based on the original conditional claims. They are not contradictions, but simply are unknown.

Take this example:

If I go on vacation I will go to Hawaii

Vac -- HI

Now, I tell you that I am in Hawaii and that I must, therefore, be on vacation.

HI --> Vac

Does this conclusion contradict the original claim? Not at all. Could it be true? Sure it could! I could indeed be on vacation in Hawaii. But does my presence in Hawaii prove that I am on vacation, based solely on the prior claim? It does not. I could be there on business. I could have washed ashore after a shipwreck. Maybe I was kidnapped and brought there against my will, or I was sleepwalking and got on a plane going there, or I joined the Navy and got stationed there. No contradiction, but no proof.

One more thing about answer A, and that is that it does not even have to be a contradiction or any kind of flaw. The conclusion COULD conflict with the claims of the physicalists, without any error, if my intention in the argument was to disprove their claim. "Some physicalists say X, but they are wrong, and here's why" would be an example of such an argument. My conclusion would conflict with their claims, but that would be the whole point. That's not the issue here, of course, as our author is trying to tell us that the physicalists' predictions will soon come true, but since the conclusion does not contradict that claim but rather supports it, A cannot be the correct answer.

I hope that distinction between an answer being unsupported or unproven, like conditional errors, versus an answer actually being contradictory, proved helpful. Keep at it!
 starre
  • Posts: 28
  • Joined: Sep 27, 2016
|
#30442
Thank you, Adam! That makes so much more sense!
 chiickenx
  • Posts: 21
  • Joined: Apr 30, 2019
|
#64577
Hi, I am super confused to how (E) is is the flaw...Even if we know how neurons interact, we still cannot properly infer the conclusion. If this is the case, then (E) is tangential to the flaw -- not the flaw itself. Can you please elaborate on how (E) is a flaw?

My diagram is as follows:

Physicalists: Mental Functions will be explainable in neurobiological terms

1. Mental functions explainable in neurobiological terms → knowledge of neurons and a neuron’s basic functions
2. Mental functions explainable in neurobiological terms → knowledge of how neurons interact
3. Mental functions explainable in neurobiological terms → delineation of psychological faculties to be explained
4. Knowledge of function of neuron
5. Delineation of psychological faculties to be explained

Conclusion: Mental Functions will be explainable in neurobiological terms
With (4) and (5), we only have the necessary conditions of (1) and (2). Even if we have the necessary condition of (2), we still cannot properly infer the conclusion. If this is the case, how is (E) even a flaw at all? its irrelevant...
 James Finch
PowerScore Staff
  • PowerScore Staff
  • Posts: 943
  • Joined: Sep 06, 2017
|
#64623
Hi Chicken,

The key here is to recognize that the stimulus gives us a conditional relationship with three necessary conditions to a single sufficient condition: explaining mental functions in neurobiological terms requires knowledge of neurons and their basic functions, a knowledge of how neurons interact, and a delineation of the psychological faculties to be explained, or:

EMFNBT :arrow: KNBF + KNI + DPF

with a contrapositive of:

KNBF or KNI or DPF :arrow: EMFNBT

We're then told that we already know KNBF and DPF (2 of our 3 necessary conditions), but we know nothing about KNI. As the stimulus concludes that EMFNBT is "bound to happen in the near future," or certain, this leads to two related flaws. First, the conclusion requires reading the conditional relationship as a Mistaken Reversal of what is given in the stimulus; secondly, even if this were a biconditional and EMFNBT could be read as a sufficient condition, we still don't know anything about the third condition, KNI. Thus we can't conclude anything based on the information in the stimulus, because we don't know whether the third necessary condition is fulfilled or not. Either flaw could be an answer choice and would be correct, but only the second is present and represented by (E).

Hope this clears things up!
 blade21cn
  • Posts: 100
  • Joined: May 21, 2019
|
#72600
A quick question, will (A) never be a correct answer in a LR Flaw question, because it's not specific enough? Not pinpointing the flaw per se? As we all know, the blaring flaw with this argument is mistaken reversal. Can't we interpret (A) to mean the conclusion contradicts the physicalists' claim in that it took the necessary condition as a sufficient condition? The dictionary definition of "contradict" is: assert the opposite of a statement made by (someone). Thanks!

Get the most out of your LSAT Prep Plus subscription.

Analyze and track your performance with our Testing and Analytics Package.